- PowerScore Staff
- Posts: 5972
- Joined: Mar 25, 2011
- Fri Mar 05, 2021 1:18 pm
#84918
Complete Question Explanation
(The complete setup for this game can be found here: lsat/viewtopic.php?t=6605)
The correct answer choice is (B)
The question stem establishes that Q and S both run in the meet, and from the first rule we can infer that T must also run in the meet, immediately after Q. Because the second rule limits S to running in just the first or third races, let’s examine what could occur in each scenario:
If S runs in the first race, then R could run in the second race, and Q and T could run in the
third and fourth races (S-R-Q-T). While this is just one solution with S running in the first
race, it proves that S can run first.
If S runs in the third race, then the QT block must run in the first two races. U then runs in
the fourth race (Q-T-S-U). This proves that Q can run in the first race.
As we have explored all possibilities for who can be first based on the possible positions of S, we can determine that only Q and S can run first, and thus answer choice (B) is correct.
(The complete setup for this game can be found here: lsat/viewtopic.php?t=6605)
The correct answer choice is (B)
The question stem establishes that Q and S both run in the meet, and from the first rule we can infer that T must also run in the meet, immediately after Q. Because the second rule limits S to running in just the first or third races, let’s examine what could occur in each scenario:
If S runs in the first race, then R could run in the second race, and Q and T could run in the
third and fourth races (S-R-Q-T). While this is just one solution with S running in the first
race, it proves that S can run first.
If S runs in the third race, then the QT block must run in the first two races. U then runs in
the fourth race (Q-T-S-U). This proves that Q can run in the first race.
As we have explored all possibilities for who can be first based on the possible positions of S, we can determine that only Q and S can run first, and thus answer choice (B) is correct.
Dave Killoran
PowerScore Test Preparation
Follow me on X/Twitter at http://twitter.com/DaveKilloran
My LSAT Articles: http://blog.powerscore.com/lsat/author/dave-killoran
PowerScore Podcast: http://www.powerscore.com/lsat/podcast/
PowerScore Test Preparation
Follow me on X/Twitter at http://twitter.com/DaveKilloran
My LSAT Articles: http://blog.powerscore.com/lsat/author/dave-killoran
PowerScore Podcast: http://www.powerscore.com/lsat/podcast/